La siguiente prueba se basa en el artículo "Onto Endomorphisms are Isomorphisms", Amer. Math. Monthly 78 (1971), 357-362 de Morris Orzech, Universidad de Queen.
KCd me habló de este documento en este hilo .
La idea es reducir el teorema a un caso fácil en el que $A$ es un anillo noetheriano.
Lema (una ligera generalización del ejercicio 6.1 de Atiyah-Macdonald) Dejemos que $A$ sea un anillo no necesariamente conmutativo. Sea $M$ sea un noetheriano $A$ -módulo. Sea $N$ ser un $A$ -submódulo de $M$ . Dejemos que $f\colon N \rightarrow M$ sea una suryectiva $A$ -homorfismo. Entonces $f$ es inyectiva.
Prueba Sea $K_n = \ker(f^n)$ , $n = 1, 2,\dots$ . Desde $M$ es noetheriano, existe $n$ tal que $K_n = K_{n+1} = \cdots$ . Dejemos que $x \in K_1$ . Desde $f$ es suryente, existe $x_2, \dots, x_n$ tal que
$x = f(x_2)$
$x_2 = f(x_3)$
$\dots$
$x_{n-1} = f(x_n)$
$x_n = f(x_{n+1})$
Desde $x_{n+1} \in K_{n+1}$ , $x_{n+1} \in K_n$ . Por lo tanto, $f^n(x_{n+1}) = 0$ . Por lo tanto, $x = f(x_2) = f^2(x_3) = \cdots = f^n(x_{n+1}) = 0$ . QED
Teorema (una generalización del teorema de Vasconcelos) . Dejemos que $A$ sea un anillo conmutativo. Sea $M$ sea una entidad finitamente generada $A$ -módulo. Sea $N$ ser un $A$ -submódulo de $M$ . Dejemos que $f\colon N \rightarrow M$ sea una suryectiva $A$ -homorfismo. Entonces $f$ es inyectiva.
Prueba Sea $0 \neq y_0 \in N$ . Basta con demostrar $f(y_0) \neq 0$ . Sea $f(y_0) = x_0$ .
Dejemos que $x_1, \dots, x_n$ ser generadores de $M$ . Sea $f(y_i) = x_i$ , $i = 1,\dots, n$ .
Supongamos que $f(x_i) = \sum_{j = 1}^{n} a_{i, j} x_j, i = 0, 1,\dots, n$ y $y_i = \sum_{j = 1}^{n} b_{i, j} x_j, i = 0, 1,\dots, n$ .
Dejemos que $B = \mathbb{Z}[a_{i, j}, b_{i, j}]$ . $B$ es un subring noetheriano de $A$ .
Dejemos que $P = Bx_1 + \cdots + Bx_n$ , $Q = By_0 + By_1 + \cdots + By_n$ . Desde $y_i \in P, i = 0, 1, ..., n, Q \subset P$ . Desde $f(y_i) = \sum_{j=1}^{n} b_{i, j} f(x_j) \in P, f(Q) \subset P$ . Por lo tanto, $f$ induce una $B$ -homorfismo $g\colon Q \rightarrow P$ . Desde $f(y_i) = x_i, i = 1,\dots, n$ , $g$ es proyectiva. Por lo tanto, por el lema, $g$ es inyectiva. Por lo tanto, $f(y_0) = g(y_0) \neq 0$ como se desee. QED
6 votos
Puede que quieras leer el artículo de la wikipedia sobre los amplificadores de clase D
4 votos
Si usas un lm386 como amplificador, la calidad de audio va a ser pobre a pesar de todo
2 votos
@PlasmaHH ... o esto Nota de la aplicación Maxim sobre amplificadores de clase D .
0 votos
Para un sistema de onda cuadrada de frecuencia no constante, busca la modulación delta-sigma o sigma-delta. Funciona para ADCs y DACs y puede ser un sistema casi completamente digital para implementar un DAC.
1 votos
1A) ¿Cómo puede una única señal analógica contener múltiples frecuencias sumadas simultáneamente? 1B) ¿Cómo puede el PWM aproximar una única señal analógica?
0 votos
Esto puede ser de interés. Para mí es una de las explicaciones más claras de cómo funciona esto: romanblack.com/BTc_alg.htm
0 votos
Fuertemente relacionado con sus preguntas 2/3: electronics.stackexchange.com/questions/12345/ - ver especialmente electrónica.stackexchange.com/a/12358/20088 para una respuesta indirecta.